0 Daumen
3,1k Aufrufe

ich würde gerne die ∑(von k=1 bis n)k in expliziter Form vorliegen haben.

Mit HERLEITUNG bitte!!!!

Avatar von

Wenn du die Summenformel  ∑(von k=1 bis n)k^0  kennst (die ist einfach), kannst du daraus durch Indexverschiebung die für k^1 (also  ∑(von k=1 bis n)k )  und daraus wiederum deine gesuchte für k^2 herleiten.

Hier eine andere etwas spielerische Methode vom enthusiastischsten Mathematiker auf ganz YouTube:


5 Antworten

+2 Daumen
 
Beste Antwort

Schau z.B. mal unter

http://was-die-welt-im-innersten-zusammenhaelt.de/eine-einfache-herleitung-der-summe-von-quadratzahlen/

Ich hatte mal eine noch etwas leichtere Herleitung, die ich momentan aber nicht finde.

Avatar von 477 k 🚀
+2 Daumen

$$\begin{aligned} a_{n+1}-a_1 &=  \sum_{i=1}^n \left(a_{i+1}-a_i\right) \qquad |~ a_i = i^3 \cr                      \left(n+1\right)^3-1^3 &=  \sum_{i=1}^n \left(\left(i+1\right)^3-i^3\right) \cr                        \left(n+1\right)^3-1 &=  \sum_{i=1}^n \left(3i^2+3i+1\right) \cr                        \left(n+1\right)^3-1 &= 3*\sum_{i=1}^n \left(i^2\right)+{3n\left(n+1\right) \over 2}+n \cr              n^3+{3\over 2}n^2+{1 \over 2}n &= 3*\sum_{i=1}^n i^2 \cr {n\left(n+1\right)\left(2n+1\right) \over 6} &= \sum_{i=1}^n i^2 \cr \end{aligned}$$

Hinweis:

Äußerst leistungsfähig ist auch der Operatorenkalkül, welcher aber leider weitgehend in Vergessenheit geraten ist.

Avatar von
+2 Daumen

Aloha :)

Da du eine HERLEITUNG möchtest, gehe ich davon aus, dass du die fertige Summenformel noch nicht kennst und daher keine vollständige Induktion zum Beweis anwenden kannst. Wir leiten zunächst die Summenformel für \(\sum_{k=1}^nk\) her. Dazu betrachten wir:$$(k-1)^2=k^2-2k+1\quad\Rightarrow\quad \underbrace{k^2-(k-1)^2}_{=:a_k}=\underbrace{2k-1}_{=:b_k}$$Die Summe über die linke Seite des Gleichheitszeichens können wir mit Indexverschiebung berechnen:$$\sum\limits_{k=1}^na_k=\sum\limits_{k=1}^n\left(k^2-(k-1)^2\right)=\sum\limits_{k=1}^nk^2-\sum\limits_{k=1}^n(k-1)^2=\sum\limits_{k=1}^nk^2-\sum\limits_{k=0}^{n-1}k^2=n^2$$Die Summe der rechten Seite ist einfacher:$$\sum\limits_{k=1}^nb_k=\sum\limits_{k=1}^n(2k-1)=2\sum\limits_{k=1}^nk-\sum\limits_{k=1}^n1=2\sum\limits_{k=1}^nk-n$$Weil alle Summanden gleich sind \((a_k=b_k)\), sind auch die beiden Summen gleich:$$2\sum\limits_{k=1}^nk-n=n^2\quad\Rightarrow\quad2\sum\limits_{k=1}^nk=n^2+n\quad\Rightarrow\quad\underline{\sum\limits_{k=1}^nk=\frac{n^2+n}{2}}$$

Jetzt wenden wir genau dasselbe Prinzip für die Summenformel von \(\sum_{k=1}^nk^2\) an:$$(k-1)^3=k^3-3k^2+3k-1\quad\Rightarrow\quad \underbrace{k^3-(k-1)^3}_{=:a_k}=\underbrace{3k^2-3k+1}_{=:b_k}$$Die Summe der linken Seite berechnen wir wieder mit Indexverschiebung:$$\sum\limits_{k=1}^na_k=\sum\limits_{k=1}^n\left(k^3-(k-1)^3\right)=\sum\limits_{k=1}^nk^3-\sum\limits_{k=1}^n(k-1)^3=\sum\limits_{k=1}^nk^3-\sum\limits_{k=0}^{n-1}k^3=n^3$$Die Summe der rechten Seite ist dank der Vorarbeit von oben:$$\sum\limits_{k=1}^nb_k=\sum\limits_{k=1}^n(3k^2-3k+1)=3\sum\limits_{k=1}^nk^2-3\sum\limits_{k=1}^nk+\sum\limits_{k=1}^n1=3\sum\limits_{k=1}^nk^2-3\frac{n^2+n}{2}+n$$Weil alle Summanden gleich sind \((a_k=b_k)\), sind auch die beiden Summen gleich:$$3\sum\limits_{k=1}^nk^2-3\frac{n^2+n}{2}+n=n^3$$$$3\sum\limits_{k=1}^nk^2=n^3+3\frac{n^2+n}{2}-n=n^3+\frac{3n^2}{2}+\frac{n}{2}$$$$\sum\limits_{k=1}^nk^2=\frac{n^3}{3}+\frac{n^2}{2}+\frac{n}{6}=\frac{2n^3+3n^2+n}{6}=\frac{n(2n^2+3n+1)}{6}$$$$\underline{\sum\limits_{k=1}^nk^2=\frac{n(n+1)(2n+1)}{6}}$$

Avatar von 148 k 🚀
+1 Daumen
Mit HERLEITUNG bitte!!!!

Ansatz:

Pyramidenförmigen Turm aus Quadratcentimeterwürfelchen bauen.

Summanden sind quadratische Flächen übereinander.

Du baust von oben nach unten.

Näherungsweise ergibt sich als Volumen der Pyramide mit Höhe n:

V = 1/3 * n^2 * n = 1/3 n^3

Das nun geeignet korrigieren.

Z.B. Ansatz für Summe Nr.n  S(n) kubisches Polynom

S(n) = a n^3 + b n^2 + cn + d

a wird vermutlich 1/3 sein. Aber du kannst einfach mal ein paar S(n) auszählen und so a, b, c, und d berechnen.

Avatar von 162 k 🚀
+1 Daumen
Avatar von 39 k

Sorry war der falsche Link, gilt für $$   \sum_{k=1}^\infty \frac{1}{k^2} = \frac{\pi^2}{6}  $$

Aber hier

https://www.mathelounge.de/133771/summenformel-fur-k-und-vollstandige-induktion-mit-gauss-n-2

Die Frage ist dann ob ein Beweis mittels der vollständigen Induktion eine Herleitung ist.

Da ist ja nicht die Vollständige Induktion bewiesen worden sondern mittels Vollständiger Induktion das verlangte Ergebnis. Mathematisch doch einwandfrei.

Ich habe das sprachlich korrigiert. Ich denke ein Beweis ist keine Herleitung.

Ein anderes Problem?

Stell deine Frage

Willkommen bei der Mathelounge! Stell deine Frage einfach und kostenlos

x
Made by a lovely community